Difference between revisions of "2008 AMC 10B Problems/Problem 19"

m (Problem)
(Video Solution(Based on Solution 1))
(9 intermediate revisions by 8 users not shown)
Line 1: Line 1:
 
==Problem==
 
==Problem==
 
A cylindrical tank with radius <math>4</math> feet and height <math>9</math> feet is lying on its side. The tank is filled with water to a depth of <math>2</math> feet. What is the volume of water, in cubic feet?
 
A cylindrical tank with radius <math>4</math> feet and height <math>9</math> feet is lying on its side. The tank is filled with water to a depth of <math>2</math> feet. What is the volume of water, in cubic feet?
 +
 +
<math>
 +
\mathrm{(A)}\ 24\pi - 36 \sqrt {2}
 +
\qquad
 +
\mathrm{(B)}\ 24\pi - 24 \sqrt {3}
 +
\qquad
 +
\mathrm{(C)}\ 36\pi - 36 \sqrt {3}
 +
\qquad
 +
\mathrm{(D)}\ 36\pi - 24 \sqrt {2}
 +
\qquad
 +
\mathrm{(E)}\ 48\pi - 36 \sqrt {3}
 +
</math>
 +
 +
==Video Solution(Based on Solution 1)==
 +
https://www.youtube.com/watch?v=I9RRrumPRK4
  
 
==Solution==
 
==Solution==
{{solution}}
+
Any vertical cross-section of the tank parallel with its base looks as follows:
 +
<asy>
 +
unitsize(0.8cm);
 +
defaultpen(0.8);
 +
pair s=(0,0), bottom=(0,-4), mid=(0,-2);
 +
pair x[]=intersectionpoints( (-10,-2)--(10,-2), circle(s,4) );
 +
fill( arc(s,x[0],x[1]) -- cycle, lightgray );
 +
draw( circle(s,4) );
 +
dot(s);
 +
draw( s -- bottom );
 +
label( "$2$", (mid+bottom)/2, E );
 +
draw ( s -- x[0] -- x[1] -- s );
 +
label( "$4$", (s+x[0])/2, NW );
 +
label( "$4$", (s+x[1])/2, NE );
 +
label( "$A$", s, N );
 +
label( "$B$", x[0], W );
 +
label( "$C$", x[1], E );
 +
label( "$D$", mid, NW );
 +
label( "$E$", bottom, S );
 +
</asy>
 +
 
 +
The volume of water can be computed as the height of the tank times the area of the shaded part.
 +
 
 +
Let <math>\theta</math> be the size of the smaller angle <math>DAC</math>. We then have <math>\cos\theta = \frac{AD}{AC}=\frac 12</math>, hence <math>\theta=60^\circ</math>.
 +
 
 +
Thus the angle <math>CAB</math> has size <math>360^\circ - 2\cdot 60^\circ = 120^\circ</math>. Hence the non-shaded part consists of <math>\frac{120^\circ}{360^\circ} = \frac 13</math> of the circle, minus the area of the triangle <math>ABC</math>.
 +
 
 +
Using the [[Pythagorean theorem]] we can compute that <math>CD=\sqrt{AC^2-AD^2}=\sqrt{16-4}=2\sqrt 3</math>. Thus <math>BC=4\sqrt 3</math>, and the area of the triangle <math>ABC</math> is <math>\frac {2 \cdot 4\sqrt 3} 2 = 4\sqrt 3</math>.
 +
 
 +
The area of the shaded part is then <math>\frac{4^2\pi}3 - 4\sqrt 3</math>, and the volume of water is <math>9\cdot\left(\frac{4^2\pi}3 - 4\sqrt 3\right) = \boxed{48\pi - 36\sqrt 3}</math>. The answer is <math>\text{E}</math>.
 +
 
 +
Alternatively, we can solve for DC and see it is congruent to a 30 60 90 triangle by SSS ~Williamgolly
  
 
==See also==
 
==See also==
 
{{AMC10 box|year=2008|ab=B|num-b=18|num-a=20}}
 
{{AMC10 box|year=2008|ab=B|num-b=18|num-a=20}}
 +
{{MAA Notice}}

Revision as of 12:20, 28 December 2020

Problem

A cylindrical tank with radius $4$ feet and height $9$ feet is lying on its side. The tank is filled with water to a depth of $2$ feet. What is the volume of water, in cubic feet?

$\mathrm{(A)}\ 24\pi - 36 \sqrt {2} \qquad \mathrm{(B)}\ 24\pi - 24 \sqrt {3} \qquad \mathrm{(C)}\ 36\pi - 36 \sqrt {3} \qquad \mathrm{(D)}\ 36\pi - 24 \sqrt {2} \qquad \mathrm{(E)}\ 48\pi - 36 \sqrt {3}$

Video Solution(Based on Solution 1)

https://www.youtube.com/watch?v=I9RRrumPRK4

Solution

Any vertical cross-section of the tank parallel with its base looks as follows: [asy] unitsize(0.8cm); defaultpen(0.8); pair s=(0,0), bottom=(0,-4), mid=(0,-2); pair x[]=intersectionpoints( (-10,-2)--(10,-2), circle(s,4) ); fill( arc(s,x[0],x[1]) -- cycle, lightgray ); draw( circle(s,4) ); dot(s); draw( s -- bottom ); label( "$2$", (mid+bottom)/2, E ); draw ( s -- x[0] -- x[1] -- s ); label( "$4$", (s+x[0])/2, NW ); label( "$4$", (s+x[1])/2, NE ); label( "$A$", s, N ); label( "$B$", x[0], W ); label( "$C$", x[1], E ); label( "$D$", mid, NW ); label( "$E$", bottom, S ); [/asy]

The volume of water can be computed as the height of the tank times the area of the shaded part.

Let $\theta$ be the size of the smaller angle $DAC$. We then have $\cos\theta = \frac{AD}{AC}=\frac 12$, hence $\theta=60^\circ$.

Thus the angle $CAB$ has size $360^\circ - 2\cdot 60^\circ = 120^\circ$. Hence the non-shaded part consists of $\frac{120^\circ}{360^\circ} = \frac 13$ of the circle, minus the area of the triangle $ABC$.

Using the Pythagorean theorem we can compute that $CD=\sqrt{AC^2-AD^2}=\sqrt{16-4}=2\sqrt 3$. Thus $BC=4\sqrt 3$, and the area of the triangle $ABC$ is $\frac {2 \cdot 4\sqrt 3} 2 = 4\sqrt 3$.

The area of the shaded part is then $\frac{4^2\pi}3 - 4\sqrt 3$, and the volume of water is $9\cdot\left(\frac{4^2\pi}3 - 4\sqrt 3\right) = \boxed{48\pi - 36\sqrt 3}$. The answer is $\text{E}$.

Alternatively, we can solve for DC and see it is congruent to a 30 60 90 triangle by SSS ~Williamgolly

See also

2008 AMC 10B (ProblemsAnswer KeyResources)
Preceded by
Problem 18
Followed by
Problem 20
1 2 3 4 5 6 7 8 9 10 11 12 13 14 15 16 17 18 19 20 21 22 23 24 25
All AMC 10 Problems and Solutions

The problems on this page are copyrighted by the Mathematical Association of America's American Mathematics Competitions. AMC logo.png